After landing on an unexplored Klingon planet, Spock tests for the direction of the magnetic field by firing a beam of electrons in various directions and by recording the following observations:1. Electrons moving upward feel a magnetic force in the northwest direction.2. Electrons moving horizontally toward the north are pushed downward.3. Electrons moving horizontally toward the southeast are pushed upward.Mr. Spock therefore concludes that the magnetic field at this landing site is in which direction?A) toward the eastB) toward the northeastC) toward the southwestD) toward the southeastE) toward the west

Answers

Answer 1

Answer:

the correct one is C,

Explanation:

The magnetic force is given by the expression

        F = q v x B

where the bold letters indicate vectors, we can find the modulus of this expression

         F = q v B sin θ

The direction of the to the force with the rule of the right hand. The thumb applies in the direction of the speed, the fingers extended in the direction of the magnetic field and if the charge is positive the palm is in the direction of the force, if the charge is megative the force is in the opposite direction to the palm of hand.

Let's analyze the different situations presented; As the particles emitted are electrons, their charge is negative.

Situation 1. The thumb points upwards, the force is towards the NW as the charge is negative, the palm points towards the SE therefore the finger points towards the SOUTHWEST which must be the direction of the magnetic field

Situation 2. the electrons go horizontally to the north the thumb points in this direction, the force is downwards, therefore the palm points upwards, the extended fingers point to the WEST

Situation 3. the thumb points to the SE towards where the electrons move, the force is upwards the palm points to the SW, therefore the fingers extended points SOUTHWEST

As a result of situations 1 and 3 the magnetic field must point in the SOUTHWEST direction.

Situation 2 gives a different result because the component of the field that is directed to the south does not create exerts and therefore cannot be detected.

When checking the answers, the correct one is C


Related Questions

Chromosomes _____.

carry thousands of genes
can be analyzed in a karyotype
always occur in pairs
are tight coils of DNA
are carried on genes

Answers

Answer:

The correct answer for this would be D.

Explanation:

Are tight coils of DNA

The bus lay 40 km at a speed of 72 km / h, and then another 60 km at a speed of 30 m / s. Determine the average speed of the bus along the way.

Answers

Answer:

25 m/s

Explanation:

From the question,

Average speed = Total distance /total time.

S' = D/T........................... Equation 1

D = 40+60 = 100 Km = 100000 m.

T = t₁+t₂

t₁ = (40×3600/72) s = 2000 s

t₂ = 60000/30 = 2000 s

T = 2000+2000 = 4000 s.

SUbstitute the values of T and D into equation 1

S' = 100000/4000

S' = 25 m/s

Using Bernoulli's equation Calculate the percentage increase in the speed of the air on the upper surface of the wings relative to the lower surface,if aircraft mass is 3.3×10^5kg,Total wing area is 500m^2 ,Flight level speed of 9600km/h and density of air is 1.2kgm^-3??​

Answers

Explanation:

3d7eei4rueiuxx7ex5x775xe

NEED HELP What color is the container for R-134a refrigerant? A. Light blue B. Yellow C. Dark green D.White and yellow​

Answers

Answer:

It is A. Light blue

_____________

Hope this helps!

┈┏━╮╭━┓┈╭━━━━╮

┈┃┏┗┛┓┃╭┫o i n k    ┃

┈╰┓▋▋┏╯╯╰━━━━╯

╭━┻╮╲┗━━━━╮╭╮┈

┃▎▎┃╲╲╲╲╲╲┣━╯┈

╰━┳┻▅╯╲╲╲╲┃┈┈┈

┈┈╰━┳┓┏┳┓┏╯┈┈┈

┈┈┈┈┗┻┛┗┻┛┈┈┈┈

You have two cylindrical bar magnets with the same mass, length, and dipole moment, but different moments of inertia, I1 and I2. You place each bar magnet so that it is initially oriented perpendicular to a uniform magnetic field. When you let each bar magnet go from rest, you find that magnet 1 takes less time than magnet 2 to rotate so that it is parallel to the magnetic field. Is I1 greater than, less than, or equal to I2

Answers

Answer:

[tex]l1 < l2[/tex];  [tex]l1[/tex] is less than [tex]l2[/tex]

Explanation:

Firstly, we take a look at the concept of moment of inertia, This concept comes from the fact that it resists change in motion. A body that has a higher value of moment of inertia requires more force ( torque) to change its position as compared to a body with lesser value of moment of inertia.

Now taking a look at the situation in  the question, both the magnets are placed in similar magnetic field, so having the same amount of force, hence torque acts on both magnets.

Since magnet 1 takes less time to rotate compared to magnate 2, hence magnet 1's moment of inertia should be less than that of magnet 2.

Therefore, [tex]l1 < l2[/tex];  [tex]l1[/tex] is less than [tex]l2[/tex]

The desperate contestants on a TV survival show are very hungry. The only food they can see is some fruit hanging on a branch high in a tree. Fortunately, they have a spring they can use to launch a rock. The spring constant is 900 N/m, and they can compress the spring a maximum of 45 cm. All the rocks on the island seem to have a mass of 500 g.
a) With what speed does the rock leave the spring?
b) To what height can the rock be launched?

Answers

Answer:

a) v = 18.86 m / s, b)  h = 8.85 m

Explanation:

a) For this exercise we can use the conservation of energy relations.

Starting point. Like the compressed spring

          Em₀ = K_e + U = ½ k x² + m g x

the zero of the datum is placed at the point of the uncompressed spring

Final point. With the spring if compress

           Em_f = K = ½ m v²

how energy is conserved

          Em₀ = Em_f

          ½ k x² + m g x = ½ m v²

   

           v² = [tex]\frac{k}{m}[/tex]  x² + 2gx

let's reduce the magnitudes to the SI system

          m = 500 g = 0.500 kg

          x = -45 cm = -0.45 m

the negative sign is because the distance in below zero of the reference frame

       

let's calculate

           v² = [tex]\frac{900}{0.500}[/tex]  0.45² + 2 9.8 (- 0.45)

           v = √355.68

           v = 18.86 m / s

b) For this part we use the conservation of energy with the same initial point and as an end point at the point where the rock stops

           Em_f = U = m g h

           Em₀ = Em_f

            ½ k x²2 + m g x = m g h

            h = ½  [tex]\frac{k}{g}[/tex]   x² + x

let's calculate

           h = [tex]\frac{1}{2} \ \frac{900}{9.8 } \ 0.45^2[/tex] - 0.45

           h = 8.85 m

measured from the point where the spring is uncompressed

When an object threw to the free space to make an angle of 25 degree at an initial speed of 15 m/sec, the ball takes time to reach another position 1.12 sec. Find the travelling path of the object.

Answers

Answer:

The horizontal distance traveled by the projectile is 15.23 m.

Explanation:

Given;

angle of projection, θ = 25⁰

initial velocity of the projectile, u = 15 m/s

time of flight, t = 1.12 s

The the travelling path of the object is calculated as the range of the projectile

[tex]R = u_x t\\\\R = (15\ Cos \ 25^0) \times 1.12\\\\R = 13.595 \times 1.12\\\\R = 15.23 \ m[/tex]

Therefore, the horizontal distance traveled by the projectile is 15.23 m.

Two bicycle tires are set rolling with the same initial speed of 4.0 m/s along a long, straight road, and the distance each travels before its speed is reduced by half is measured. One tire is inflated to a pressure of 40 psi and goes a distance of 18.2 m; the other is at 105 psi and goes a distance of 92.6 m. Assume that the net horizontal force is due to rolling friction only and take the free-fall acceleration to be g = 9.80 m/s2. What is the coefficient of rolling friction μr for the tire under low pressure?

Answers

Answer:

The coefficient of rolling friction will be "0.011".

Explanation:

The given values are:

Initial speed,

[tex]v_i = 4.0 \ m/s[/tex]

then,

[tex]v_f=\frac{4.0}{2}[/tex]

    [tex]=2.0 \ m/s[/tex]

Distance,

s = 18.2 m

The acceleration of a bicycle will be:

⇒ [tex]a=\frac{v_f^2-v_i^2}{2s}[/tex]

On substituting the given values, we get

⇒    [tex]=\frac{(2.0)^2-(4.0)^2}{2\times 18.2}[/tex]

⇒    [tex]=\frac{4-8}{37}[/tex]

⇒    [tex]=\frac{-4}{37}[/tex]

⇒    [tex]=0.108 \ m/s^2[/tex]

As we know,

⇒  [tex]f=ma[/tex]

and,

⇒  [tex]\mu_rmg=ma[/tex]

⇒       [tex]\mu_r=\frac{a}{g}[/tex]

On substituting the values, we get

⇒       [tex]=\frac{0.108}{9.8}[/tex]

⇒       [tex]=0.011[/tex]

Which scale do geologists use to estimate the total energy released by an earthquake?

Answers

Answer:

Richter scale Magnitude

Richter scale. Magnitude is a measure of the amount of energy released during an earthquake, and you've probably heard news reports about earthquake magnitudes measured using the Richter scale. Something like, "A magnitude 7.3 earthquake struck Japan today.

Explanation:

Hope I helped!

Help please thank you

Answers

I believe the answer would be C because potential energy is affected by height and mass. The truck in photo C is the highest and has a lot of mass.

compare and contrast series and parallel circuits?

Answers

In a series circuit, a common current flows through all the components of the circuit. While in a parallel circuit, a different amount of current flows through each parallel branch of the circuit. Whereas in the parallel circuit, the same voltage exists across the multiple components in the circuit.

Hope It Helps!

A 1,250 kg car is moving due to 6,500 N engine force. If the kinetic friction coefficient between the car and the road is 0.32, what is the car's acceleration?


A) 32m/s²

B) 200m/s²

C) 50m/s²

D) 2m/s²






Answer and I will give you brainiliest

Answers

Answer:

b i hope this is correct answee

Three 5 Ohm resistors are connected in parallel to a 9 Volt power supply. What is the current through each resistor?

Answers

Answer:

I₁ = I₂ = I₃ = 12 A

Explanation:

Since the resistors are connected in parallel across a battery. Hence, the voltage across each of them will be the same, which will be equal to the voltage of the power supply. Since each resistor has the same resistance of 5 Ω. Therefore. the current across each resistor will also be the same. We can easily calculate these current values by the use of Ohm's Law, as follows:

[tex]V = IR\\I_{1}=I_{2}=I_{3}=\frac{V}{R} \\\\I_{1}=I_{2}=I_{3}=\frac{9\ Volt}{5 \Omega}\\\\[/tex]

I₁ = I₂ = I₃ = 12 A

A cart moves with negligible friction or air resistance along a roller coaster track. The cart starts from rest at the top of a hill of unknown height. It then goes down that first hill and up another circularly shaped hill of height 10 meters and radius 15 meters. The ride is designed so that the cart just barely loses contact with the track at the top of the second hill (10 meters above ground level). What is the height of the starting hill relative to ground

Answers

Answer:

hinit = 17.5 m

Explanation:

Assuming no friction present, the mechanical energy must be conserved, which means that at any point of the trajectory, the sum of the gravitational potential energy and the kinetic energy must keep the same.At the top of the hill, since it starts from rest, all the energy must be potential, and we can express it as follows:

       [tex]E_{o} = U_{o} = m*g*h_{init} (1)[/tex]

When the car arrives to the top of the second hill, as we know that it is lower than the first one, the energy of the car, must be part gravitational potential energy, and part kinetic energy.We can express this final energy as follows:

       [tex]E_{f} = U_{f} + K_{f} = m*g* h_{2} + \frac{1}{2} *m*v_{f} ^{2} (2)[/tex]

In order to find hinit, we need to make (1) equal to (2), and solve for it.In (2) we have the value of h₂ (10 m), but we still need the value of the speed at the top of the second hill, vf.Now, when the car is at the top of the hill, there are two forces acting on it, in opposite directions: the normal force (upward) and the weight (downward).We know also that there is a force that keeps the car along the circular track, which is the centripetal force.This force is just the net downward force acting on the car (it's vertical at the top), and is just the difference between the weight and the normal force.If the cart just barely loses contact with the track at the top of the second hill, this means that at that point the normal force becomes zero.So, the centripetal force must be equal to the weight.The centripetal force can be expressed as follows:

       [tex]F_{c} = m*\frac{v_{f} ^{2}}{R} (3)[/tex]

We have just said that (3) must be equal to the weight:

       [tex]F_{c} = m*\frac{v_{f} ^{2}}{R} = m*g (4)[/tex]

Simplifying, and rearranging, we can solve for vf², as follows:

       [tex]v_{f}^{2} = R*g (5)[/tex]  

Replacing (5) in (2), simplifying and rearranging in (1) and (2) we finally have:

      [tex]h_{init} = h_{2} + \frac{1}{2} R = 10m + 7.5 m = 17.5 m (6)[/tex]

A kicker punts a football from the very center of the field to the sideline 41 yards downfield. (A football field is 53 yards wide.)What is the angle between the direction of the net displacement of the ball and the 50-yard line of the field

Answers

Answer:

Ur answer is 12 sir!

Explanation:

HAve a good day sir

If you lean against a wall, the wall pushes back on you with a weaker force

true
false

Answers

false - (.do not ) if you lean against a wall the wall pushes back on you with a( n) weaker force

Please help
A meteoroid with a mass of 1,000 kg enters Earth's atmosphere with a velocity of 4,400 m/s. What is the magnitude of the meteorold's momentum at
that instant?

Answers

answer is 4,400,000 kg • m/s

Forces act on a box that is initially at rest as shown. Coefficient of friction between the object and the surface is μ=0.1.

If the box' mass is 6kg box and if it is moving with acceleration of 0.4m/s2, what distance will the box travel if it was pushed for 4s?

Answers

Answer:

3.2 m

Explanation:

From the question given above, the following data were obtained:

Coefficient of friction (μ) = 0.1

Initial velocity (u) = 0 m/s

Mass (m) = 6 Kg

Acceleration (a) = 0.4 m/s²

Time (t) = 4 s

Distance (s) =.?

The distance travelled can be obtained as illustrated below:

s = ut + ½at²

s = (0×4) + (½ × 0.4 × 4²)

s = 0 + (0.2 × 16)

s = 0 + 3.2

s = 3.2 m

Thus, the box will travel 3.2 m in 4 s.

27. The traffic officer issued violation tickets to traffic
violators. If 68 of them composed 80% of the total
violators, how many violators are there in all?
A. 83
B. 84
C. 85
D. 86
28. There are 18 roses in a bunch of 24 flowers.
What percent of the flowers are roses?
A. 18%
B. 24% C. 60% D. 75%

Please help me ❤️❤️ thank u
Hope god​

Answers

Answer:

27: 85

28:75%

Explanation:

27:68=80

?=100 hence (68×100)÷80

=85

28:18/24× 100

=75%

The provided text reads: "a typical lightning bolt may transfer 10^20 electrons in a fraction of a second, developing a peak current of up to 10 kiloamperes." Using the value of the elementary charge of 1.6x10^-19 C We can estimate the total charge of the lightning bolt to be about *
1.6E1 C
1.6E4 C
1.6E10 C
1.6E20 C

Answers

Answer:

1.6e20

Explanation:

Please please help I'm stuck!!!

Answers

Answer: iDc im in 3grade

Explanation:sorry

I GOTCHU THE ANSWER IS 2.44s

A uniform 140 g rod with length 57 cm rotates in a horizontal plane about a fixed, vertical, frictionless pin through its center. Two small 30 g beads are mounted on the rod such that they are able to slide without friction along its length. Initially the beads are held by catches at positions 11 cm on each sides of the center, at which time the system rotates at an angular speed of 23 rad/s. Suddenly, the catches are released and the small beads slide outward along the rod. Find the angular speed of the system at the instant the beads reach the ends of the rod. Answer in units of rad/s.

Answers

Answer:

The correct answer is "12 rad/s"

Explanation:

The given values are,

Mass of rod,

M = 140 g

i.e.,

   = 0.14 kg

Length,

L = 57 cm

i.e.,

  = 0.57 m

Mass of beads,

M = 30 g

i.e.,

   = 0.03 kg

Angular speed,

r = 11 cm

i.e.,

 = 0.11 m

Now,

The inertia of rods will be:

=  [tex]\frac{1}{12}ML ^2[/tex]

On substituting the values, we get

=  [tex]\frac{1}{12}\times 0.14\times (0.57)^2[/tex]

=  [tex]0.0037905 \ kg-m^2[/tex]

The inertia of beads will be:

=  [tex]mr^2[/tex]

On substituting the values, we get

=  [tex]0.03\times (0.11)^2[/tex]

=  [tex]0.000726 \ kg-m^2[/tex]

The total inertia will be:

=  [tex]Inertia \ of \ rods+Inertia \ of \ beads[/tex]

=  [tex]0.0037905 + 0.000726[/tex]

=  [tex]0.0045165 \ kg-m^2[/tex]

now,

The angular speed of the system will be:

⇒ [tex]L_1w_1=L_2w_2[/tex]

On substituting the values in the above equation, we get

⇒ [tex]0.0045165\times 23 = (0.0037905 + (0.03\times 0.285^2)\times 2 )\times w_2[/tex]

⇒ [tex]0.1038795 = 0.0037905 + (0.00243675\times 2 )\times w_2[/tex]

⇒             [tex]w_2 = 12 \ rad/s[/tex]

PLEASEEEE HELP ASAP
What are the two main divisions
of the peripheral nervous system?
A. Muscular Nervous System, Digestive Nervous
System
B. Circulatory Nervous System, Endocrine
Nervous System
C. Somatic Nervous System, Autonomic Nervous
System

Answers

It’s divided into two parts, the somatic nervous system and the autonomic nervous system.

hope this helps!! :)

Alternate current

Hello!
I need solve 2 questions, I tried but im so bad at physics :(

1. If the frequency in the network is 50Hz, what is the period of this voltage?
2. The effective voltage value in the network is 230V. What is its maximum value?

I would like an explanation with a calculation for my understanding!!

Thanks in advance you all!

Answers

sorry bro but i don't know his i am also bad

Explanation:

but maybe you can you the formula of volatage as well as

What is optics???????

see my percentage of answering and thanks , it's crazy ​

Answers

Answer:

hi..bro

make pinky sis answer as brinlist ok

have a good day

An 80 N force causes a spring to compress 0.15 m. What is the spring constant? What is the potential energy of the spring?

Answers

The spring constant of the given spring is= -533.33

The potential energy of the spring= 6 Joule

What is force?

A push or pull that causes object to change its state of rest or motion.

It can also change the direction of a moving body.Force is a product of mass and acceleration.

For spring;

K is the spring constant.

         ⇒Force = -kx

         ⇒80N=0.15m ×-k

          ⇒K=-80/0.15

          ⇒K= -533.333

So the spring constant is 533.333

Potential Energy=1/2kx²

                            ⇒0.5×533.33 ×0.15×0.15

                            ⇒5.95.99 N

Learn more about force here: http://brainly.com/question/25239010

#SPJ2

What is the name of the supercontinent that developed during the Paleozoic?

Laurasia
Tethys
Pangaea
Rodinia

Answers

Pangea trust me I’ve done this

A force of 100N moves a body on a horizontal frictionless surface when......

I. Friction force exerted on the body is less than 100N
II. Friction force exerted on the body is more than 100N
III. None of these
IV. Both of I and II

Answers

Answer:

I. Friction force exerted on the body is less than 100N

Explanation:

For  a body to be static, the moving force must be equal to the frictional force. Since the frictional force is a force of opposition. It tends to oppose the moving force acting on an object.

Hence if the moving force is greater than the force of friction, the Force of fiction will not be able to overcome the moving hence the body will tend to move.

Therefore, for a body to move, Fm > Ff or Ff < Ff

Fm is the moving force

Ff is the force of friction

Given

Fm = 100N

For the 100N body to move the frictional force must be less than 100N

Which of the following is a conductor?

copper
water
aluminum
all of the above

Answers

Answer:

D all of the above

Explanation:

electricity moves easily through all of them and none of them prevent the flow of electricity

Gaseous sulfur dioxide is a compound that combines with water in the atmosphere to form acid rain. What is the primary source of sulfur dioxide?
O volcanic emissions
O combustion of fossil fuels
O destruction of tropical forests
O mining and mineral extraction

Answers

The answer is combustion of fossil fuels.
Other Questions
During Hartsfields time in office, Atlanta became known as "A City Too Busy to Hate. What factors supported this nickname? Check all that apply.The city integrated its schools.Atlanta became a leader in aviation.The economy declined.The population increased to one million.Many people served as the city mayor.Its, A,B,D Someone please!!!!!! give me the correct answer to this question.I will mark you brainliest for the correct answer.This is a big test.Please help!!!! PLEASE HURRYYYY ITS DUE SOON 4, 5, 3, 3, 1, 2, 3, 2, 4, 8, 2, 4, 4, 5, 2, 3, 6,2Find the mean, median, and mode(s) of the data. Which measure bestrepresents the data? Explain your reasoning.Soon this is due in 5 minutes find area and perimeter Water molecules are formed by covalentbonding. What effect does this have onthe water molecule?creates a polar molecule with two positiveendscreates a nonpolar molecule with two positiveendscreates a nonpolar molecule with one positiveand one negative endcreates a polar molecule with one positiveand one negative end Solve a0 = 0, a1 = 1 and an = an1 + an2 + 2n , for n 2. Hints: This one involves a lot more algebra than the problems above. Solving the homogeneous problem will involve using the quadratic formula. The characteristic roots turn out to be 1 5 2 (but show the work!). For a particular solution, try an = A2 n . You should find A = 4 (but show the work!). Put those two pieces together to write down the general solution an = A( (1 + 5 )/2 )^n + B( (1 5)/ 2 )^n + 4(2^n ) and the determine the values for A and B by using the two initial conditions, a0 = 0 and a1 = 1. The necessary arithmetic will be somewhat complicated, but not impossible. Jennifer has 14 coins that total $2.15. She only has dimes and quarters. If she onlyhas dimes and quarters, how many dimes does she have The first, third and thirteenth terms of an arithmetic sequence are the first 3 terms of a geometric sequence. If the first term of both sequences is 1, determine:1.) the first three terms of the geometric sequence if r > 12.) the sum of 7 terms of the geometric sequence if the sequence is 1, 5, 25 The Cuban Revolution in the 1950s led to the end of the trade relationship between which country and Cuba? A) Spain B) England C) The Soviet union D) The United States Use Substitution to solve the following System of Equations.y= 4x + 3y = -2 2 In both Mesopotamia and Ancient China, the early civilization startedA. in high mountainsB. near two rivers systemsC. on the ocean shoreD. around three big lakes physics!!! please help D: Should cabinet members be selected by American citizens or the president? While researching an argumentative essay on livestock and the Columbian Exchange, Salim finds a .org websiterun by a group who believes that all animals should live in the wild and not be used to help humans. Why shouldSalim avoid using this source?The information on the site is likely not updated.The information on the site can be edited by anyone.The site might feature authors who are not experts on wild horses.O The site might have bias against using horses to move people and goods. I need help simplifying this equation. Drag tiles to match each fraction, decimal, or percent to an equivalent number.24/25 3/5 9/50 26% 0.350.96 60% 0.18 13/50 7/20 PLEASE ANSWER ASAP Which of these is a binomial? O A. x + 2y2 - 7 O B. ab O C. 5xy O D. 2x3 - 7y3 List and explain ONE request that Don Corleone heard or granted as a favor. This comes from the movie Godfather part 1. Can someone please help me